Brilliant Summation Contest Season-1 (Part 1)

Hi Brilliant! I've decided to start the first ever summation contest here.

The aims of the Summation contest are to improve skills in the computation of sums, to learn from each other as much as possible, and of course to have fun. Anyone here may participate in this contest.

The rules are as follows:

  • I will start by posting the first problem. If there is a user solves it, then they must post a new one.

  • You may only post a solution of the problem below the thread of problem and post your proposed problem in a new thread. Put them separately.

  • Only make substantial comment that will contribute to the discussion.

  • Make sure you know how to solve your own problem before posting it in case there is no one can answer it within 48 hours, then you must post the solution and you have a right to post another problem.

  • If the one who solves the last problem does not post his/her own problem after solving it within a day, then the one who has a right to post a problem is the last solver before him/her.

  • You are NOT allowed to post a multiple summation problem.

  • Problems must be purely of summation. They shouldn't have integrals and products in them. Solutions can follow methods that use integrals and products.

  • It is NOT compulsory to post original problems. But make sure it has not been posted on brilliant.

  • There is no restriction in the standard of summations.

Please post your solution and your proposed problem in a single new thread.

Format your post is as follows:

1
2
3
4
5
6
7
**SOLUTION OF PROBLEM xxx (number of problem) :**

**[Post your solution here]**

**PROBLEM xxx (number of problem) :**

**[Post your problem here]**

The comments will be easiest to follow if you sort by "Newest":

See Part-2.

#Calculus

Note by Aditya Kumar
4 years, 11 months ago

No vote yet
1 vote

  Easy Math Editor

This discussion board is a place to discuss our Daily Challenges and the math and science related to those challenges. Explanations are more than just a solution — they should explain the steps and thinking strategies that you used to obtain the solution. Comments should further the discussion of math and science.

When posting on Brilliant:

  • Use the emojis to react to an explanation, whether you're congratulating a job well done , or just really confused .
  • Ask specific questions about the challenge or the steps in somebody's explanation. Well-posed questions can add a lot to the discussion, but posting "I don't understand!" doesn't help anyone.
  • Try to contribute something new to the discussion, whether it is an extension, generalization or other idea related to the challenge.
  • Stay on topic — we're all here to learn more about math and science, not to hear about your favorite get-rich-quick scheme or current world events.

MarkdownAppears as
*italics* or _italics_ italics
**bold** or __bold__ bold

- bulleted
- list

  • bulleted
  • list

1. numbered
2. list

  1. numbered
  2. list
Note: you must add a full line of space before and after lists for them to show up correctly
paragraph 1

paragraph 2

paragraph 1

paragraph 2

[example link](https://brilliant.org)example link
> This is a quote
This is a quote
    # I indented these lines
    # 4 spaces, and now they show
    # up as a code block.

    print "hello world"
# I indented these lines
# 4 spaces, and now they show
# up as a code block.

print "hello world"
MathAppears as
Remember to wrap math in \( ... \) or \[ ... \] to ensure proper formatting.
2 \times 3 2×3 2 \times 3
2^{34} 234 2^{34}
a_{i-1} ai1 a_{i-1}
\frac{2}{3} 23 \frac{2}{3}
\sqrt{2} 2 \sqrt{2}
\sum_{i=1}^3 i=13 \sum_{i=1}^3
\sin \theta sinθ \sin \theta
\boxed{123} 123 \boxed{123}

Comments

Problem 2: Prove that n1(1)nn2(γ+ψ(3n+12))=632ζ(3)+ζ(2)log214πG\displaystyle \sum_{n\geq 1} \frac{(-1)^n}{n^2}\left(\gamma+\psi\left(3n+\frac12\right) \right)=\frac{63}{2}\zeta(3)+\zeta(2)\log 2 -14\pi G

GG is Catalan's constant.

This problem has been solved by Ishan Singh.

Proposition:

n1(1)nn2(γ+ψ(kn+12))=72k2ζ(3)+2πj=12k1jCl2(πj/k+π2k)πj=1k1jCl2(2πj/k+π/k)+ζ(2)log2\displaystyle \sum_{n\geq 1} \frac{(-1)^n}{n^2}\left(\gamma+\psi\left(kn+\frac12\right) \right)=\frac72k^2\zeta(3)+2\pi\sum_{j=1}^{2k-1}j\text{Cl}_2(\pi j/k+\frac{\pi}{2k}) - \pi\sum_{j=1}^{k-1}j\text{Cl}_2(2\pi j/k + \pi/k) + \zeta(2)\log 2

Aman Rajput - 4 years, 11 months ago

Log in to reply

Solution Of Problem 2 :

Proposition : n=1Hnxnn2=Li3(x)Li3(1x)+ln(1x)Li2(1x)+12lnxln2(1x)+ζ(3) \sum_{n=1}^{\infty} \dfrac{H_{n} x^n}{n^2} = \operatorname{Li_{3}} (x) - \operatorname{Li}_{3} (1-x) + \ln(1-x) \operatorname{Li}_{2}(1-x) + \dfrac{1}{2} \ln x \ln^2 (1-x) + \zeta(3)

Proof : It is easy to see that

n=1Hnxnn=Li2(x)+12ln2(1x) \displaystyle \sum_{n=1}^{\infty} \dfrac{H_{n} x^n}{n} = \operatorname{Li}_{2}(x) + \dfrac{1}{2} \ln^2(1-x)

Dividing by xx and integrating, we have,

n=1Hnxnn2=Li2(x)xdx+12ln2(1x)xdx \displaystyle \sum_{n=1}^{\infty} \dfrac{H_{n} x^n}{n^2} = \int \dfrac{\operatorname{Li}_{2}(x)}{x}\mathrm{d}x + \dfrac{1}{2} \int \dfrac{\ln^2(1-x)}{x}\mathrm{d}x

=Li3(x)+12[ln2(1x)lnx]+lnxln(1x)1xdx = \operatorname{Li}_{3}(x) + \dfrac{1}{2} [\ln^2(1-x) \ln x] + \int \dfrac{\ln x \ln (1-x)}{1-x}\mathrm{d}x

Let x=1tx=1-t

    n=1Hnxnn2=Li3(x)+12[ln2(1x)lnx]ln(1t)lnttdt \implies \displaystyle \sum_{n=1}^{\infty} \dfrac{H_{n} x^n}{n^2} = \operatorname{Li}_{3}(x) + \dfrac{1}{2} [\ln^2(1-x) \ln x] - \int \dfrac{\ln (1-t) \ln t}{t} \mathrm{d}t

Now,

ln(1t)lnttdt=lntLi2(t)Li2(t)tdt\int \dfrac{\ln (1-t) \ln t}{t} \mathrm{d}t = \ln t \operatorname{Li}_{2}(t) - \int \dfrac{ \operatorname{Li}_{2}(t) }{t}\mathrm{d}t

=lntLi2(t)Li3(t) = \ln t \operatorname{Li}_{2}(t) - \operatorname{Li}_{3}(t)

Substituting back, we have,

n=1Hnxnn2=Li3(x)Li3(1x)+ln(1x)Li2(1x)+12lnxln2(1x)+C \displaystyle \sum_{n=1}^{\infty} \dfrac{H_{n} x^n}{n^2} = \operatorname{Li_{3}} (x) - \operatorname{Li}_{3} (1-x) + \ln(1-x) \operatorname{Li}_{2}(1-x) + \dfrac{1}{2} \ln x \ln^2 (1-x) + C

Putting x=0x=0, we get C=ζ(3)C=\zeta(3), thus,

n=1Hnxnn2=Li3(x)Li3(1x)+ln(1x)Li2(1x)+12lnxln2(1x)+ζ(3) \sum_{n=1}^{\infty} \dfrac{H_{n} x^n}{n^2} = \operatorname{Li_{3}} (x) - \operatorname{Li}_{3} (1-x) + \ln(1-x) \operatorname{Li}_{2}(1-x) + \dfrac{1}{2} \ln x \ln^2 (1-x) + \zeta(3) \quad \square

Now,

S=n=1(1)nn2(γ+ψ(3n+12))=2ln2n=1(1)nn2+n=1k=13n22k1\displaystyle \text{S} = \sum_{n = 1}^{\infty} \frac{(-1)^n}{n^2}\left(\gamma+\psi\left(3n+\frac{1}{2} \right) \right) = -2\ln 2 \sum_{n = 1}^{\infty} \dfrac{(-1)^n}{n^2} + \sum_{n = 1}^{\infty} \sum_{k=1}^{3n} \dfrac{2}{2k-1}

Also,

k=13n12k1=k=16n1kk=13n12k\sum_{k=1}^{3n} \dfrac{1}{2k-1} = \sum_{k=1}^{6n} \dfrac{1}{k} - \sum_{k=1}^{3n}\dfrac{1}{2k}

k=13n12k1=H6n12H3n \sum_{k=1}^{3n} \dfrac{1}{2k-1} = H_{6n} - \dfrac{1}{2} H_{3n}

    S=ζ(2)ln2+n=1(1)nn2[2H6nH3n] \implies \text{S} = \zeta(2) \ln 2 + \sum_{n = 1}^{\infty} \dfrac{(-1)^n}{n^2} [2 H_{6n} - H_{3n}]

Using cube roots and sixth roots of unity on the proposition, we have,

n=1(1)nn2[2H6nH3n]=632ζ(3)14πG \sum_{n = 1}^{\infty} \dfrac{(-1)^n}{n^2} [2 H_{6n} - H_{3n}] = \dfrac{63}{2} \zeta(3) -14 \pi G

    S=ζ(2)ln2+632ζ(3)14πG\implies \text{S} = \boxed{\zeta(2) \ln 2 + \dfrac{63}{2} \zeta(3) -14 \pi G}

Ishan Singh - 4 years, 11 months ago

PROBLEM 11

Evaluate n=1(1)nHn2n+1  . \sum_{n=1}^\infty (-1)^n \frac{H_n}{2n+1} \;.

This problem has been solved by Jake Lai and Aditya Kumar.

Mark Hennings - 4 years, 11 months ago

Log in to reply

It is well-known that ln(1x)1x=n=1Hnxn\displaystyle -\frac{\ln(1-x)}{1-x} = \sum_{n=1}^\infty H_nx^n. So

n=1(1)nHn2n+1=01n=1Hn(x2)n dx=01ln(1+x2)1+x2 dx(Substitute u=tan1x)=0π/4ln(1+tan2u) du=0π/4ln(sec2u) du=0π/42lncosu du=Gπ2ln2\begin{aligned} \sum_{n=1}^\infty \frac{(-1)^nH_n}{2n+1} &= \int_0^1 \sum_{n=1}^\infty H_n(-x^2)^n \ dx \\ &= \int_0^1 \frac{-\ln(1+x^2)}{1+x^2} \ dx & (\text{Substitute } u = \tan^{-1} x) \\ &= \int_0^{\pi/4} -\ln(1+\tan^2 u) \ du \\ &= \int_0^{\pi/4} -\ln(\sec^2 u) \ du \\ &= \int_0^{\pi/4} 2 \ln \cos u \ du \\ &= G - \frac{\pi}{2} \ln 2 \end{aligned}

where G is Catalan's constant.

Jake Lai - 4 years, 11 months ago

Log in to reply

Reversing the order of integration and summation to obtain the second line of your argument takes a bit of justification. The generating function series is not monotonic, nor is it uniformly bounded by an integrable function, so the result is not automatic.

You need to integrate from 00 to x<1x < 1, obtaining N=1(1)nHn2n+1x2n+1  =  20tan1xlncosudu \sum_{N=1}^\infty \frac{(-1)^n H_n}{2n+1} x^{2n+1} \; =\; 2\int_0^{\tan^{-1}x} \ln \cos u\,du and then let xx tend to 11-, using Abel's Theorem to get the answer (having used the Alternating Series Test to prove that the series n(1)nHn2n+1\sum_n (-1)^n\frac{H_n}{2n+1} is convergent).

Anyway, you are up for the next question!

Mark Hennings - 4 years, 11 months ago

Alternate Method:

n=1(1)n2n+1Hn=n=12H2n4n+1n=1Hn2n+1\sum _{ n=1 }^{ \infty }{ \frac { { \left( -1 \right) }^{ n } }{ 2n+1 } { H }_{ n } } =\sum _{ n=1 }^{ \infty }{ \frac { 2{ H }_{ 2n } }{ 4n+1 } } -\sum _{ n=1 }^{ \infty }{ \frac { { H }_{ n } }{ 2n+1 } }

Then we can use appropriate generating functions for evaluating it. The integrals have to be carefully handled in order to get the solution.

Aditya Kumar - 4 years, 11 months ago

SOLUTION OF PROBLEM 1 :

n0(1)nψ(3)(2n+2)=3!n0(1)nk01(k+2n+2)4=6n,k0(1)n(k+2n+2)4  =  6K0(n=0K/2(1)n)1(K+2)4=6K0,K0,1  (4)1(K+2)4=38K01(2K+1)4+6K01(4K+3)4=38(1116)190π4+3128ζ(4,32)=1256π4+1256ψ(3)(34) \begin{array}{rcl} \displaystyle \sum_{n \ge 0} (-1)^n \psi^{(3)}(2n+2) & = & \displaystyle 3!\sum_{n \ge 0} (-1)^n\sum_{k \ge 0}\frac{1}{(k+2n+2)^4} \\ & = & \displaystyle 6\sum_{n,k \ge 0} \frac{(-1)^n}{(k+2n+2)^4} \; = \; 6\sum_{K \ge 0} \left(\sum_{n=0}^{\lfloor K/2 \rfloor} (-1)^n\right) \frac{1}{(K+2)^4} \\ & = & \displaystyle 6\sum_{K \ge 0\,,\,K \equiv 0,1 \; (4)} \frac{1}{(K+2)^4} \\ & = & \displaystyle \tfrac38\sum_{K \ge 0} \frac{1}{(2K+1)^4} + 6\sum_{K \ge 0} \frac{1}{(4K+3)^4} \\ & = & \displaystyle \tfrac38\big(1 - \tfrac{1}{16}\big)\tfrac{1}{90}\pi^4 + \tfrac{3}{128}\zeta(4,\tfrac32) \\ & = & \tfrac{1}{256}\pi^4 + \tfrac{1}{256}\psi^{(3)}\big(\tfrac34\big) \end{array}

Mark Hennings - 4 years, 11 months ago

Special Problem :

Evaluate

r=0n(1)r(nr) \sum_{r=0}^{n} \dfrac{(-1)^r}{\dbinom{n}{r}}

Ishan Singh - 4 years, 11 months ago

Log in to reply

Solution to Special Problem : Method 1\underline{\text{Method 1}}

S=r=0n(1)r(nr)\text{S}=\displaystyle\sum_{r=0}^{n}\dfrac{(-1)^r}{\dbinom{n}{r}}

=r=0n(1)rr!(nr)!n!=\displaystyle\sum_{r=0}^{n}(-1)^r\dfrac{r!(n-r)!}{n!}

=r=0n(1)rr!(nr)!(nr+1+r+1)n!×1(n+2)=\displaystyle\sum_{r=0}^{n}(-1)^r \dfrac{r!(n-r)!\color{#3D99F6}{(n-r+1+r+1)}}{n!}\color{#D61F06}{\times\dfrac{1}{(n+2)}}

=1n!(n+2)r=0n[(1)rr!(nr+1)!+(1)r(r+1)!(nr)!]=\displaystyle \dfrac{1}{n!(n+2)} \sum_{r=0}^{n} \left[(-1)^r r!(n-r+1)! + (-1)^r (r+1)!(n-r)! \right]

=1n!(n+2)r=0n[(1)rr!(nr+1)!(1)r+1(r+1)!(nr)!]=\displaystyle \dfrac{1}{n!(n+2)} \sum_{r=0}^{n} \left[(-1)^r r!(n-r+1)! - (-1)^{r+1} (r+1)!(n-r)! \right]

=1n!(n+2)r=0n(TrTr+1)(*)\color{#3D99F6}{=\displaystyle \dfrac{1}{n!(n+2)} \sum_{r=0}^{n} \left( T_{r} - T_{r+1} \right) }\tag{*}

where Tr=(1)rr!(nr+1)!T_{r}= (-1)^r r!(n-r+1)!

Clearly, ()(*) is a telescoping series. Evaluating it, we have,

S=(1)nTn+1+T0\text{S}=(-1)^nT_{n+1}+T_{0}

=(1)n(n+1)!n!(n+2)+(n+1)!n!(n+2)=(-1)^n\dfrac{(n+1)!}{n!(n+2)} + \dfrac{(n+1)!}{n!(n+2)}

Since nn is even,

S=2n+1n+2\color{#D61F06}{\therefore\text{S}=\boxed{2\dfrac{n+1}{n+2}}}


Method 2\underline{\text{Method 2}}

Consider the following Lemma.

Lemma : r=0k(1)r(kr)=0\color{#624F41}{\displaystyle \sum_{r=0}^k\dfrac{(-1)^r}{\dbinom{k}{r}}=0}

where kk is an odd positive integer.

Proof : S=r=0k(1)r(kr)\text{S}=\displaystyle \sum_{r=0}^k\dfrac{(-1)^r}{\dbinom{k}{r}}

=r=0k(1)kr(kkr)=\displaystyle \sum_{r=0}^k\dfrac{(-1)^{k-r}}{\dbinom{k}{k-r}}

(r=0nf(r)=r=0nf(nr))\displaystyle\color{#3D99F6}{\left(\because \sum_{r=0}^{n} f(r) = \sum_{r=0}^{n} f(n-r) \right)}

=r=0k(1)r(kr)= -\displaystyle \sum_{r=0}^k\dfrac{(-1)^r}{\dbinom{k}{r}}

    S=S\implies \text{S}=-\text{S}

    S=0\implies \text{S}=0

This completes the proof of the Lemma.

Now, since nn is even, n+1n+1 is odd.

Using the Lemma, we have,

r=0n+1(1)r(n+1r)=0(1)\color{#3D99F6}{\displaystyle \sum_{r=0}^{n+1}\dfrac{(-1)^r}{\dbinom{n+1}{r}}=0} \tag{1}

Let,

J=r=0n(1)r(nr)(2)\color{#D61F06}{\text{J}=\displaystyle \sum_{r=0}^n\dfrac{(-1)^r}{\dbinom{n}{r}}}\tag{2}

Operating (1)+(2)(1)+(2),

    J+0=r=0n(1)r(nr)+r=0n+1(1)r(n+1r)\implies \text{J}+0=\displaystyle \sum_{r=0}^n\dfrac{(-1)^r}{\dbinom{n}{r}} + \displaystyle \sum_{r=0}^{n+1}\dfrac{(-1)^r}{\dbinom{n+1}{r}}

=r=0n(1)r(nr)+r=0n(1)r(n+1r)+(1)(n+1n+1)=\displaystyle \sum_{r=0}^n\dfrac{(-1)^r}{\dbinom{n}{r}} + \displaystyle \sum_{r=0}^{n}\dfrac{(-1)^r}{\dbinom{n+1}{r}} +\dfrac{(-1)}{\dbinom{n+1}{n+1}}

=r=0n(1)r(nr)+r=0n(1)nr(n+1nr)1(r=0nf(r)=r=0nf(nr))=\displaystyle \sum_{r=0}^n\dfrac{(-1)^r}{\dbinom{n}{r}} + \displaystyle \sum_{r=0}^{n}\dfrac{(-1)^{n-r}}{\dbinom{n+1}{n-r}} -1 \quad \displaystyle\color{#3D99F6}{\left(\because \sum_{r=0}^{n} f(r)=\sum_{r=0}^{n} f(n-r)\right)}

=r=0n(1)r(nr)+r=0n(1)r(n+1r+1)1=\displaystyle \sum_{r=0}^n\dfrac{(-1)^r}{\dbinom{n}{r}} + \displaystyle \sum_{r=0}^{n}\dfrac{(-1)^{r}}{\dbinom{n+1}{r+1}} -1

=r=0n(1)r{1(nr)+r+1n+11(nr)}1[(nr)=nr(n1r1) & (nr)=(nnr)]=\displaystyle \sum_{r=0}^n(-1)^r\left\{\dfrac{1}{\dbinom{n}{r}} + \dfrac{r+1}{n+1} \cdot \dfrac{1}{\dbinom{n}{r}}\right\} -1 \quad \displaystyle\color{#D61F06}{\left[\because \dbinom{n}{r}=\dfrac{n}{r}\dbinom{n-1}{r-1} \ \text{\&} \ \dbinom{n}{r}=\dbinom{n}{n-r} \right]}

    J=(n+2n+1)r=0n(1)r(nr)+1n+1r=0n(1)rr(nr)1\implies \text{J} = \displaystyle\left(\dfrac{n+2}{n+1}\right) \sum_{r=0}^{n}\dfrac{(-1)^r}{\dbinom{n}{r}}+ \dfrac{1}{n+1} \sum_{r=0}^{n}\dfrac{(-1)^r\cdot r}{\dbinom{n}{r}} -1

    J+1=(n+2n+1)J+1n+1r=0n(1)rr(nr)= G (let)\implies \text{J}+1 = \displaystyle\left(\dfrac{n+2}{n+1}\right) \text{J}+ \dfrac{1}{n+1} \underbrace{\color{#66f}{\sum_{r=0}^{n}\dfrac{(-1)^r\cdot r}{\dbinom{n}{r}}}}_{\huge{= \ \text{G} \ (\text{let})}}

    J+1=(n+2n+1)J+Gn+1(3) \implies \text{J}+1= \displaystyle\left(\dfrac{n+2}{n+1}\right) \text{J}+ \dfrac{\text{G}}{n+1} \tag{3}

Now,

G=r=0n(1)rr(nr)\text{G}=\displaystyle\sum_{r=0}^{n}\dfrac{(-1)^r\cdot r}{\dbinom{n}{r}}

=r=0n(1)nr(nr)(nnr)=\displaystyle\sum_{r=0}^{n}\dfrac{(-1)^{n-r}\cdot (n-r)}{\dbinom{n}{n-r}}

=r=0n(1)r(nr)(nr)=\displaystyle\sum_{r=0}^{n}\dfrac{(-1)^{r}\cdot (n-r)}{\dbinom{n}{r}}

=nr=0n(1)r(nr)r=0n(1)rr(nr)=\displaystyle n\sum_{r=0}^{n}\dfrac{(-1)^{r}}{\dbinom{n}{r}} - \sum_{r=0}^{n}\dfrac{(-1)^{r}\cdot r}{\dbinom{n}{r}}

    G=nJG\implies \text{G}=n\cdot \text{J}-\text{G}

    G=Jn2(4)\implies\color{#D61F06}{\text{G}=\dfrac{\text{J}n}{2}} \tag{4}

From (3) & (4)(3)\ \text{\&} \ (4),

    J+1=(n+2n+1)J+Jn2(n+1)\implies \text{J}+1= \displaystyle\left(\dfrac{n+2}{n+1}\right) \text{J}+ \dfrac{\text{J}n}{2(n+1)}

J=2n+1n+2\color{#20A900}{\therefore\text{J}=\boxed{2\dfrac{n+1}{n+2}}} \quad \square

Ishan Singh - 4 years, 11 months ago

Start by writing (nr)1=(n+1)01ur(1u)nrdu\displaystyle \dbinom nr ^{-1} = (n+1) \int_0^1 u^r (1-u)^{n-r} \, du .

Multiply both sides by (1)r (-1)^r and adding over rr gives:

r=0n(1)r(nr)=(n+1)01r=0n(u)r(1u)nrdu=(n+1)01[(1u)n+1+(1)nun+1]du=n+1n+2(1+(1)n)={0,n oddn+12n+4,n even \begin{aligned} \sum_{r=0}^n\dfrac{(-1)^r}{ \binom{n}{r}} &=& (n+1) \int_0^1 \sum_{r=0}^n (-u)^r (1-u)^{n-r} \, du \\ &=& (n+1) \int_0^1 \left [(1-u)^{n+1} + (-1)^n u^{n+1} \right ] \, du \\ &=& \dfrac{n+1}{n+2} ( 1 + (-1)^n ) = \begin{cases} 0 , \qquad \quad n \text{ odd} \\ \dfrac{n+1}{2n+4} , \quad n \text{ even} \end{cases} \end{aligned}

Pi Han Goh - 4 years, 11 months ago

Log in to reply

This is the same method which Ishan talked about. Thanks for posting!

Aditya Kumar - 4 years, 11 months ago

Problem 3:
Evaluate :

r=0n[(nr)sinrxcos(nr)x]\sum_{r=0}^n \left[\binom{n}{r}\cdot\sin rx \cdot \cos (n-r)x\right]

This problem has been solved by Deeparaj Bhat and Aditya Kumar.

Ishan Singh - 4 years, 11 months ago

Log in to reply

Call the given sum SS.

Then, by using the definition of trigonometric functions via complex exponentials, we have S=14ir=0n(nr)(einxeinx+e(2rn)ixe(2rn)ix) S = \frac{1}{4i} \sum_{r=0}^{n} {n \choose r} \left( e^{inx}-e^{-inx} + e^{(2r-n)ix} - e^{-(2r-n)ix} \right)

Now, using binomial theorem and a bit of simplification, we get that S=2n1sin(nx) S=2^{n-1} \sin (nx)

PS: I don't have good summation problems. So, anyone can post in my behalf. :)

A Former Brilliant Member - 4 years, 11 months ago

Log in to reply

Alternate Solution:

We use the identity: r=0nf(r)=r=0nf(nr)\displaystyle \sum _{ r=0 }^{ n }{ f\left( r \right) } =\sum _{ r=0 }^{ n }{ f\left( n-r \right) }

We take:

S=r=0n[(nr)sinrxcos(nr)x](1) S=\sum_{r=0}^n \left[\binom{n}{r}\cdot\sin rx \cdot \cos (n-r)x\right] \quad \quad (1)

On applying the identity, we get

S=r=0n[(nnr)sin(nrx)cosrx](2) S=\sum_{r=0}^n \left[\binom{n}{n-r}\cdot\sin (n-rx) \cdot \cos rx\right] \quad \quad (2)

On adding equations 1 and 2, we get:

2S=r=0n(nr)sinnx2S=\sum_{r=0}^n \binom{n}{r} \sin nx

Therefore, S=2n1sinnxS=2^{n-1} \sin nx

Aditya Kumar - 4 years, 11 months ago

Problem 6 :

Prove That

r=1n(1)r1r(nr)=Hn\sum_{r=1}^n \frac{(-1)^{r-1}}r \dbinom{n}{r} =H_{n}

Notation : HnH_{n} denotes the Harmonic Number.

This problem has been solved by Aditya Kumar.

Ishan Singh - 4 years, 11 months ago

Log in to reply

Consider the binomial expansion:

r=0n(x)r(nr)=(1x)n\sum_{r=0}^n (-x)^{r} \dbinom{n}{r} =\left(1-x\right)^{n}

We'll write it as:

r=1n(x)r(nr)=(1x)n1\sum_{r=1}^n (-x)^{r} \dbinom{n}{r} =\left(1-x\right)^{n}-1

On dividing both the sides by x-x, we get:

r=1n(x)r1(nr)=1(1x)nx\sum_{r=1}^n (-x)^{r-1} \dbinom{n}{r} =\frac{1-\left(1-x\right)^{n}}{x}

On integrating both the sides w.r.t. xx from 0 to 1, we get:

r=1n(1)r1r(nr)=011(1x)nxdx=011xn1xdx=Hn\sum_{r=1}^n \frac{(-1)^{r-1}}r \dbinom{n}{r} =\int _{ 0 }^{ 1 }{ \frac { 1-{ \left( 1-x \right) }^{ n } }{ x } dx } =\int _{ 0 }^{ 1 }{ \frac { 1-{ x }^{ n } }{ 1-x } dx } ={ H }_{ n }\\

Aditya Kumar - 4 years, 11 months ago

Solution To Problem 6 :

S=r=1n(1)r1r(nr) \text{S} = \sum_{r=1}^{n} \dfrac{(-1)^{r-1}}{r} \dbinom{n}{r}

Note that, k=rn(k1r1)=(nr) \displaystyle \sum_{k=r}^{n} \dbinom{k-1}{r-1} = \dbinom{n}{r} , it follows,

S=r=1n(1)r1rk=rn(k1r1)\text{S} = \sum_{r=1}^{n} \dfrac{(-1)^{r-1}}{r} \sum_{k=r}^{n} \dbinom{k-1}{r-1}

Since (k1r1)=rk(kr) \displaystyle \dbinom{k-1}{r-1} = \dfrac{r}{k} \dbinom{k}{r} , we have,

S=r=1nk=rn(1)r1k(kr) \text{S} = \sum_{r=1}^{n} \sum_{k=r}^{n} \dfrac{(-1)^{r-1}}{k} \dbinom{k}{r}

Interchanging order of summation, we have,

S=k=1nr=1k(1)r1k(kr)\text{S} = \sum_{k=1}^{n} \sum_{r=1}^{k} \dfrac{(-1)^{r-1}}{k} \dbinom{k}{r}

=k=1n1k = \sum_{k=1}^{n} \dfrac{1}{k}

=Hn = H_{n} \quad \square

Ishan Singh - 4 years, 11 months ago

Problem 12:

Prove the following for x<427|x| < \dfrac{4}{27}.

n=0(3nn)xn=2cos(13sin1(33x2))427x\sum_{n=0}^\infty \binom{3n}{n}x^n = \frac{2\cos(\frac{1}{3} \sin^{-1}(\frac{3\sqrt{3x}}{2}))}{\sqrt{4-27x}}

This problem has been solved by Mark Hennings and Ishan Singh.

Jake Lai - 4 years, 11 months ago

Log in to reply

Solution to Problem 12:

By the multiplication formula for the Gamma function, (3n)!  =  Γ(3n+1)  =  12π32n+12Γ(n+13)Γ(n+23)Γ(n+1) (3n)! \; = \; \Gamma(3n+1) \; = \; \tfrac{1}{2\pi} 3^{2n+\frac12} \Gamma(n+\tfrac13)\Gamma(n+\tfrac23)\Gamma(n+1) and hence (3nn)  =  33n+122π×Γ(n+13)Γ(n+23)Γ(2n+1)  =  32n+122πB(n+23,n+13) {3n \choose n} \; = \; \frac{3^{3n+\frac12}}{2\pi} \times \frac{\Gamma(n+\frac13)\Gamma(n+\frac23)}{\Gamma(2n+1)} \; = \;\frac{3^{2n+\frac12}}{2\pi}B(n+\tfrac23,n+\tfrac13) Thus S  =  n=0(3nn)xn=32πn=032nxn01un13(1u)n23du=32π01u13(1u)23127xu(1u)du  =  32π01u13(1u)2314sin23αu(1u)du \begin{array}{rcl}\displaystyle S \; =\; \sum_{n=0}^\infty {3n \choose n}x^n & = & \displaystyle\frac{\sqrt{3}}{2\pi}\sum_{n=0}^\infty 3^{2n} x^n \int_0^1 u^{n-\frac13}(1-u)^{n-\frac23}\,du \\ & = & \displaystyle \frac{\sqrt{3}}{2\pi} \int_0^1 \frac{u^{-\frac13}(1-u)^{-\frac23}}{1 - 27xu(1-u)}\,du \; =\; \frac{\sqrt{3}}{2\pi}\int_0^1 \frac{u^{-\frac13}(1-u)^{-\frac23}}{1 - 4\sin^23\alpha\, u(1-u)}\,du \end{array} where 0<α<16π0 < \alpha < \tfrac16\pi is such that sin3α=1227x\sin3\alpha = \tfrac12\sqrt{27x}. The substitutions u=sin2θu = \sin^2\theta and then x=tanθx = \tan\theta yield S=32π012πsin23θcos43θ×2sinθcosθ1sin23αsin22θdθ  =  3π012πtan13θ1sin23αsin22θdθ=3π0x13(1+x2)(1+x2)24sin23αx2dx  =  3π0x13(1+x2)(x2+2xsin3α+1)(x22xsin3α+1)dx=3π0x13f(x)dx  =  3πI \begin{array}{rcl} S & = & \displaystyle \frac{\sqrt{3}}{2\pi} \int_0^{\frac12\pi} \frac{\sin^{-\frac23}\theta \cos^{-\frac43}\theta \times 2\sin\theta \cos\theta}{1 - \sin^23\alpha\, \sin^22\theta}\,d\theta \; = \; \displaystyle \frac{\sqrt{3}}{\pi}\int_0^{\frac12\pi} \frac{\tan^{\frac13}\theta}{1 - \sin^23\alpha\, \sin^22\theta}\,d\theta \\ & = & \displaystyle\frac{\sqrt{3}}{\pi} \int_0^\infty \frac{x^{\frac13}(1 + x^2)}{(1 + x^2)^2 - 4\sin^23\alpha x^2}\,dx \; = \; \displaystyle\frac{\sqrt{3}}{\pi} \int_0^\infty \frac{x^{\frac13}(1 + x^2)}{(x^2 + 2x \sin3\alpha + 1)(x^2 - 2x \sin3\alpha + 1)}\,dx \\ & = & \displaystyle \frac{\sqrt{3}}{\pi} \int_0^\infty x^{\frac13}f(x)\,dx \; = \; \frac{\sqrt{3}}{\pi}I \end{array} where the meromorphic function f(z)  =  z2+1(z2+2zsin3α+1)(z22zsin3α+1) f(z) \; =\; \frac{z^2 + 1}{(z^2 + 2z \sin3\alpha + 1)(z^2 - 2z \sin3\alpha + 1)} has simple poles at each of the points in R={±ie3iα,±ie3iα}\mathcal{R} \,=\, \{ \pm ie^{3i\alpha}, \pm i e^{-3i\alpha}\}.

Cutting the complex plane along the positive real axis, and integrating around the "keyhole contour" γ1+γ2γ3γ4\gamma_1 + \gamma_2 - \gamma_3 - \gamma_4 where

  • γ1\gamma_1 is the straight line segment from ϵ\epsilon to RR, just above the cut,
  • γ2\gamma_2 is the circular segment z=Reiθz = Re^{i\theta} for 0θ2π0 \le \theta \le 2\pi,
  • γ3\gamma_3 is the straight line segment from ϵe2πi\epsilon e^{2\pi i} to Re2πiR e^{2 \pi i}, just below the cut,
  • γ4\gamma_4 is the circular segment z=ϵeiθz = \epsilon e^{i\theta} for 0θ2π0 \le \theta \le 2\pi

for any 0<ϵ<1<R0 < \epsilon < 1 < R, then (γ1+γ2γ3γ4)z13f(zdz  =  2πiuRResz=uz13f(z) \left(\int_{\gamma_1} + \int_{\gamma_2} - \int_{\gamma_3} - \int_{\gamma_4}\right) z^{\frac13} f(z\,dz \; = \; 2\pi i \sum_{u \in \mathcal{R}} \mathrm{Res}_{z=u} z^{\frac13}f(z) so that (1ω)ϵRx13f(x)dx+(γ2γ4)z13f(z)dz  =  2πiuRResz=uz13f(z) (1 - \omega)\int_\epsilon^R x^{\frac13}f(x)\,dx + \left(\int_{\gamma_2} - \int_{\gamma_4}\right) z^{\frac13}f(z)\,dz \; = \; 2\pi i\sum_{u \in \mathcal{R}} \mathrm{Res}_{z=u} z^{\frac13}f(z) Letting ϵ0\epsilon \to 0 and RR \to \infty, we obtain (1ω)I  =  2πiuRResz=uz13f(z) (1 - \omega)I \; = \; 2\pi i\sum_{u \in \mathcal{R}} \mathrm{Res}_{z=u} z^{\frac13}f(z) and hence I  =  π3(2cos2α1)  =  π3(4cos2α3)  =  πcosα3cos3α I \; = \; \frac{\pi}{\sqrt{3}(2\cos2\alpha - 1)} \; = \; \frac{\pi}{\sqrt{3}(4\cos^2\alpha - 3)} \; = \; \frac{\pi \cos\alpha}{\sqrt{3}\cos3\alpha} so that S  =  cosαcos3α  =  2cos(13sin1(33x2))427x S \; = \; \frac{\cos\alpha}{\cos3\alpha} \; = \; \frac{2\cos\left(\frac13\sin^{-1}\left(\frac{3\sqrt{3x}}{2}\right)\right)}{\sqrt{4 - 27x}} as required.

Mark Hennings - 4 years, 11 months ago

Solution To Problem 12 :

Proposition : f(a,z)=0xzx2+2ax+1dx=πsinπzsin((1z)cos1(a))sin(cos1(a)) f(a,z) = \int_{0}^{\infty} \dfrac{x^z}{x^2 +2ax +1} \mathrm{d}x = \dfrac{\pi}{\sin \pi z} \dfrac{\sin((1-z) \cos^{-1}(a))}{\sin(\cos^{-1}(a))}

Proof : Note that,

n=0Un(a)(x)n=1x2+2ax+1 \sum_{n=0}^{\infty}{{U}_{n}(a) {(-x)}^{n}} = \dfrac{1}{x^2 +2ax+1}

where Un(x) U_{n} (x) is the Chebyshev Polynomial of the second kind.

    n=0Un(a)Γ(n+1)(x)nn!=1x2+2ax+1 \implies \sum_{n=0}^{\infty}{{U}_{n}(a) \Gamma(n+1) \frac{{(-x)}^{n}}{n!}} = \frac{1}{x^2 +2ax +1}

Using Ramanujan Master Theorem, we have,

f(a,z)=πsinπzUs(a) f(a,z) = \dfrac{\pi}{\sin \pi z} U_{-s} (a)

=πsinπzsin((1z)cos1(a))sin(cos1(a)) = \dfrac{\pi}{\sin \pi z} \dfrac{\sin((1-z) \cos^{-1}(a))}{\sin(\cos^{-1}(a))} \quad \square

Now, using Gamma Triplication Formula,

S=n=0(3nn)xn=32π0x13(1+x2)(x2+2ax+1)(x22ax+1)dx\text{S} = \sum_{n=0}^{\infty} \dbinom{3n}{n} x^n = \dfrac{\sqrt{3}}{2 \pi} \int_0^\infty \frac{x^{\frac{1}{3}}(1 + x^2)}{(x^2 + 2ax + 1)(x^2 - 2ax + 1)} \mathrm{d}x

where a=323xa = \dfrac{3}{2} \sqrt{3x}

Using Partial Fraction and the Proposition, we have,

S=32π[12f(13,a)+12f(73,a)14af(43,a)14af(103,a)+12f(13,a)+12f(73,a)+14af(43,a)+14af(103,a)]\text{S} = \dfrac{\sqrt{3}}{2\pi} \left[ \dfrac{1}{2} f \left( \dfrac{1}{3} , -a \right) + \dfrac{1}{2} f\left( \dfrac{7}{3} , -a \right) - \dfrac{1}{4a} f\left( \dfrac{4}{3} , -a \right) - \dfrac{1}{4a} f\left( \dfrac{10}{3} , -a \right) + \dfrac{1}{2} f \left( \dfrac{1}{3} , a \right) + \dfrac{1}{2} f\left( \dfrac{7}{3} , a \right) + \dfrac{1}{4a} f\left( \dfrac{4}{3} , a \right) + \dfrac{1}{4a} f\left( \dfrac{10}{3} , a \right)\right]

After simplification, we have,

S=2cos(13sin1(33x2))427x \text{S} = \dfrac{2\cos(\frac{1}{3} \sin^{-1}(\frac{3\sqrt{3x}}{2}))}{\sqrt{4-27x}} \quad \square

Ishan Singh - 4 years, 11 months ago

PROBLEM 13:

Show that n=1(Hn(2))22n  =  1360π416π2ln22+16ln42+2Li4(12)+ζ(3)ln2 \sum_{n=1}^\infty \frac{\big(H_n^{(2)}\big)^2}{2^n} \; = \; \tfrac{1}{360}\pi^4 - \tfrac16\pi^2\ln^22 + \tfrac16\ln^42 + 2\mathrm{Li}_4(\tfrac12) + \zeta(3)\ln2

This problem has been solved by Aditya Kumar

Mark Hennings - 4 years, 11 months ago

Log in to reply

Solution to Problem 13:

Just to see how nasty the last integral in this problem can get, here are the associated indefinite integrals!

Start with B(x)=n=1Hn(2)(n+1)2xn+1  =  0x(0un=1Hn(2)vndv)duu=0x(0uLi2(v)1vdv)duu  =  0x(vxLi2(v)u(1v)du)dv=0xln(xv)Li2(v)1vdv \begin{array}{rcl} B(x) & = & \displaystyle \sum_{n=1}^\infty \frac{H_n^{(2)}}{(n+1)^2}x^{n+1} \; = \; \int_0^x \left(\int_0^u \sum_{n=1}^\infty H_n^{(2)}v^n\,dv\right)\,\frac{du}{u} \\ & = & \displaystyle \int_0^x \left(\int_0^u \frac{\mathrm{Li}_2(v)}{1-v}\,dv\right)\,\frac{du}{u} \; = \; \int_0^x \left(\int_v^x \frac{\mathrm{Li}_2(v)}{u(1-v)}\,du\right)\,dv \\ & = & \displaystyle \int_0^x \ln\big(\tfrac{x}{v}\big) \frac{\mathrm{Li}_2(v)}{1-v}\,dv \end{array} Now (differentiate back to check): 0xLi2(v)1vdv=13π2ln(1x)+ln2(1x)lnx+ln(1x)Li2(x)+2Li3(1x)2ζ(3)0xLi2(v)1vlnvdv=145π416π2ln2(1x)+14ln4(1x)+12ln2(1x)ln2x+(ln2(1x)ln(1x)lnx+ln2x)Li2(x)12Li2(x)2+ln2(x1x)Li2(x1x)+2ln(1x)Li3(1x)2lnxLi3(x)+2ln(1x)Li3(x1x)2lnxLi3(x1x)2Li4(1x)+2Li4(x)+2Li4(x1x) \begin{array}{rcl} \displaystyle \int_0^x \frac{\mathrm{Li}_2(v)}{1-v}\,dv & = & \displaystyle -\tfrac13\pi^2 \ln(1 - x) + \ln^2(1 - x) \ln x + \ln(1 - x)\mathrm{Li}_2(x) + 2\mathrm{Li}_3(1 - x) - 2 \zeta(3) \\ \displaystyle \int_0^x \frac{\mathrm{Li}_2(v)}{1-v}\ln v\,dv & = & \displaystyle \tfrac{1}{45}\pi^4 - \tfrac16 \pi^2 \ln^2(1 - x) + \tfrac14 \ln^4(1 - x) + \tfrac12\ln^2(1 - x) \ln^2x \\ & & \displaystyle + (\ln^2(1 - x) - \ln(1 - x) \ln x + \ln^2x) \mathrm{Li}_2(x) - \tfrac12 \mathrm{Li}_2(x)^2 \\ & & \displaystyle + \ln^2\left(\frac{x}{1-x}\right) \mathrm{Li}_2\left(-\frac{x}{1-x}\right) + 2 \ln(1 - x) \mathrm{Li}_3(1 - x) - 2 \ln x \mathrm{Li}_3(x) \\ & & \displaystyle + 2 \ln(1 - x) \mathrm{Li}_3\left(-\frac{x}{1-x}\right) - 2 \ln x \mathrm{Li}_3\left(-\frac{x}{1 - x}\right) - 2 \mathrm{Li}_4(1 - x) \\ & & \displaystyle + 2 \mathrm{Li}_4(x) + 2 \mathrm{Li}_4\left(-\frac{x}{1 - x}\right) \end{array} and hence B(12)  =  11440π4124π2ln22+124ln42+14ζ(3)ln2 B(\tfrac12) \; = \; \tfrac{1}{1440}\pi^4 - \tfrac{1}{24}\pi^2 \ln^22 + \tfrac{1}{24}\ln^42 + \tfrac14\zeta(3)\ln 2 Now consider A(x)=n=1(Hn(2))2xn  =  n=1(Hn1(2)+1n2)2xn=n=1n=1(Hn(2))2xn+1+2n=1Hn(2)(n+1)2xn+1+Li4(x)=xA(x)+2B(x)+Li4(x) \begin{array}{rcl} \displaystyle A(x) & = & \displaystyle \sum_{n=1}^\infty \big(H_n^{(2)}\big)^2x^n \; =\; \sum_{n=1}^\infty \left(H_{n-1}^{(2)} + \tfrac{1}{n^2}\right)^2 x^n \\ & = & \displaystyle\sum_{n=1}^\infty \sum_{n=1}^\infty \big(H_n^{(2)}\big)^2 x^{n+1} + 2\sum_{n=1}^\infty \frac{H_n^{(2)}}{(n+1)^2} x^{n+1} + \mathrm{Li}_4(x) \\ & =& \displaystyle xA(x) + 2 B(x) + \mathrm{Li}_4(x) \end{array} so that n=1(Hn(2))22n=A(12)  =  4B(12)+2Li4(12)=1360π416π2ln22+16ln42+2Li4(12)+ζ(3)ln2 \begin{array}{rcl} \displaystyle \sum_{n=1}^\infty \frac{\big(H_n^{(2)}\big)^2}{2^n} & = & \displaystyle A(\tfrac12) \; =\; 4B(\tfrac12) + 2\mathrm{Li}_4(\tfrac12) \\ & = & \displaystyle \tfrac{1}{360}\pi^4 - \tfrac16\pi^2\ln^22 + \tfrac16\ln^42 + 2\mathrm{Li}_4(\tfrac12) + \zeta(3)\ln2 \end{array} as required.

Mark Hennings - 4 years, 11 months ago

Log in to reply

I have solved the indefinite integrals by using Landen's Identity.

Ishan Singh - 4 years, 11 months ago

I was wondering whether we can extend the result to other weights too?

Ishan Singh - 4 years, 11 months ago

The main integral to evaluate here is ln2(1x)lnxxdx \displaystyle \int \dfrac{\ln^2(1-x) \ln x}{x} \mathrm{d}x. Rest of the integrals follow from this using Euler's Reflection Formula and/or IBP. The integral Li2(x)1xdx \displaystyle \int \dfrac{\operatorname{Li}_{2}(x)}{1-x} \mathrm{d}x is evaluated in my solution of Problem 2.

Ishan Singh - 4 years, 11 months ago

Can you please help me with this query which is related to the above integrals?

Ishan Singh - 4 years, 11 months ago

Solution to Problem 13:

Consider the function: f(x)=n=1(Hn(2))2xn\displaystyle f\left( x \right) =\sum _{ n=1 }^{ \infty }{ { \left( { H }_{ n }^{ (2) } \right) }^{ 2 }{ x }^{ n } } .

f(x)=n=1(Hn1(2)+1n2)2xnf\left( x \right) =\sum _{ n=1 }^{ \infty }{ { \left( { H }_{ n-1 }^{ (2) }+\frac { 1 }{ { n }^{ 2 } } \right) }^{ 2 }{ x }^{ n } }

f(x)=n=1(Hn1(2))2xn+Li4(x)+2n=1Hn1(2)xnn2f\left( x \right) =\sum _{ n=1 }^{ \infty }{ { \left( { H }_{ n-1 }^{ (2) } \right) }^{ 2 }{ x }^{ n } } +{ \text{Li} }_{ 4 }(x)+2\sum _{ n=1 }^{ \infty }{ \frac { { H }_{ n-1 }^{ (2) }{ x }^{ n } }{ { n }^{ 2 } } }

On changing the summation index from nn to n+1n+1, we get:

f(x)=xf(x)+Li4(x)+2n=1Hn(2)xn+1(n+1)2(...1)f\left( x \right) =xf\left( x \right) +{ \text{Li} }_{ 4 }(x)+2\sum _{ n=1 }^{ \infty }{ \frac { { H }_{ n }^{ (2) }{ x }^{ n+1 } }{ { \left( n+1 \right) }^{ 2 } } } \quad \quad (...1)

We have to find n=1Hn(2)xn+1(n+1)2\displaystyle \sum _{ n=1 }^{ \infty }{ \frac { { H }_{ n }^{ (2) }{ x }^{ n+1 } }{ { \left( n+1 \right) }^{ 2 } } }

n=1Hn(2)xn+1(n+1)2=0x[n=1Hn(2)xn(n+1)]dx(...2)\sum _{ n=1 }^{ \infty }{ \frac { { H }_{ n }^{ (2) }{ x }^{ n+1 } }{ { \left( n+1 \right) }^{ 2 } } } =\int _{ 0 }^{ x }{ \left[ \sum _{ n=1 }^{ \infty }{ \frac { { H }_{ n }^{ (2) }{ x }^{ n } }{ { \left( n+1 \right) } } } \right] dx } \quad \quad (...2)

Now we have to find n=1Hn(2)xn(n+1)\displaystyle \sum _{ n=1 }^{ \infty }{ \frac { { H }_{ n }^{ (2) }{ x }^{ n } }{ { \left( n+1 \right) } } }

I'll use the identity: n=1Hn(2)xnn=0xLi2(t)t(1t)dt\displaystyle \sum _{ n=1 }^{ \infty }{ \frac { { H }_{ n }^{ (2) }{ x }^{ n } }{ n } } =\int _{ 0 }^{ x }{ \frac { { \text{Li} }_{ 2 }\left( t \right) }{ t\left( 1-t \right) } dt } .

Now, we differentiate w.r.t. x and get:

n=1Hn(2)xn1=Li2(x)x(1x)\sum _{ n=1 }^{ \infty }{ { H }_{ n }^{ (2) }{ x }^{ n-1 } } =\frac { { \text{Li} }_{ 2 }\left( x \right) }{ x\left( 1-x \right) }

We multiply both sides by x:

n=1Hn(2)xn=Li2(x)(1x)\sum _{ n=1 }^{ \infty }{ { H }_{ n }^{ (2) }{ x }^{ n } } =\frac { {\text{Li} }_{ 2 }\left( x \right) }{ \left( 1-x \right) }

Now, we integrate both sides w.r.t. xx from 0 to x:

n=1Hn(2)xn+1n+1=2Li3(1x)2Li2(1x)ln(1x)Li2(x)ln(1x)ln(x)(ln(1x))22ζ(3)\sum _{ n=1 }^{ \infty }{ \frac { { H }_{ n }^{ (2) }{ x }^{ n+1 } }{ n+1 } } =2{ \text{Li} }_{ 3 }\left( 1-x \right) -2{ \text{Li} }_{ 2 }\left( 1-x \right) \ln { \left( 1-x \right) } -{ \text{Li} }_{ 2 }\left( x \right) \ln { \left( 1-x \right) } -\ln { \left( x \right) } { \left( \ln { \left( 1-x \right) } \right) }^{ 2 }-2\zeta \left( 3 \right)

On dividing both sides by xx:

n=1Hn(2)xnn+1=2Li3(1x)x2Li2(1x)ln(1x)xLi2(x)ln(1x)xln(x)(ln(1x))2x2ζ(3)x\sum _{ n=1 }^{ \infty }{ \frac { { H }_{ n }^{ (2) }{ x }^{ n } }{ n+1 } } =\frac { 2{ \text{Li} }_{ 3 }\left( 1-x \right) }{ x } -\frac { 2{ \text{Li} }_{ 2 }\left( 1-x \right) \ln { \left( 1-x \right) } }{ x } -\frac { {\text{Li} }_{ 2 }\left( x \right) \ln { \left( 1-x \right) } }{ x } -\frac { \ln { \left( x \right) } { \left( \ln { \left( 1-x \right) } \right) }^{ 2 } }{ x } -\frac { 2\zeta \left( 3 \right) }{ x }

Now plugging this in eqn 22, we get:

n=1Hn(2)xn+1(n+1)2=0x[2Li3(1x)x2Li2(1x)ln(1x)xLi2(x)ln(1x)xln(x)(ln(1x))2x2ζ(3)x]dx\sum _{ n=1 }^{ \infty }{ \frac { { H }_{ n }^{ (2) }{ x }^{ n+1 } }{ { \left( n+1 \right) }^{ 2 } } } =\int _{ 0 }^{ x }{ \left[ \frac { 2{ \text{Li} }_{ 3 }\left( 1-x \right) }{ x } -\frac { 2{ \text{Li} }_{ 2 }\left( 1-x \right) \ln { \left( 1-x \right) } }{ x } -\frac { { \text{Li} }_{ 2 }\left( x \right) \ln { \left( 1-x \right) } }{ x } -\frac { \ln { \left( x \right) } { \left( \ln { \left( 1-x \right) } \right) }^{ 2 } }{ x } -\frac { 2\zeta \left( 3 \right) }{ x } \right] dx }

Now plugging this in eqn 11, we get:

f(x)=Li4(x)1x+21x0x[2Li3(1x)x2Li2(1x)ln(1x)xLi2(x)ln(1x)xln(x)(ln(1x))2x2ζ(3)x]dxf\left( x \right) =\frac { { \text{Li} }_{ 4 }\left( x \right) }{ 1-x } +\frac { 2 }{ 1-x } \int _{ 0 }^{ x }{ \left[ \frac { 2{ \text{Li} }_{ 3 }\left( 1-x \right) }{ x } -\frac { 2{\text{Li} }_{ 2 }\left( 1-x \right) \ln { \left( 1-x \right) } }{ x } -\frac { { \text{Li} }_{ 2 }\left( x \right) \ln { \left( 1-x \right) } }{ x } -\frac { \ln { \left( x \right) } { \left( \ln { \left( 1-x \right) } \right) }^{ 2 } }{ x } -\frac { 2\zeta \left( 3 \right) }{ x } \right] dx }

Now we take x=12x=\frac { 1 }{ 2 } and compute the integral. Then we finally get:

n=1(Hn(2))22n  =  1360π416π2ln22+16ln42+2Li4(12)+ζ(3)ln2\boxed{\displaystyle \sum_{n=1}^\infty \frac{\big(H_n^{(2)}\big)^2}{2^n} \; = \; \tfrac{1}{360}\pi^4 - \tfrac16\pi^2\ln^22 + \tfrac16\ln^42 + 2\mathrm{Li}_4(\tfrac12) + \zeta(3)\ln2}

The final integral is left to the reader's exercise. I didn't post it as it would've made this solution too long.

Aditya Kumar - 4 years, 11 months ago

Log in to reply

I'm impressed. The last integral is a brute.

Mark Hennings - 4 years, 11 months ago

Log in to reply

@Mark Hennings In fact we can even generalize the result by finding the indefinite integral of the last one and in turn the generating function n=1(Hn(2))2xn \displaystyle \sum_{n=1}^{\infty} (H_{n} ^{(2)})^2 x^n . The solution gets too long though.

Ishan Singh - 4 years, 11 months ago

Log in to reply

@Ishan Singh very beautiful. we need to be very very cautious.

Srikanth Tupurani - 2 years, 3 months ago

Problem 1:

Prove that: n=0(1)nψ3(2n+2)=π4256+ψ(3)(34)256.\sum_{n=0}^{\infty}(-1)^{n}\psi_{3}(2n+2) = \frac{\pi^4}{256}+\frac{\psi^{(3)}\left(\frac{3}{4} \right)}{256}.

This problem has been first solved by Aman Rajput and second by Mark Hennings.

Aditya Kumar - 4 years, 11 months ago

Solution to Problem 1:

Use the relation km+1ψ3(kz)=0nk1ψ3(z+n/k)\displaystyle k^{m+1}\psi^3(kz)=\sum_{0\leq n \leq k-1}\psi^3(z+n/k)

Which reduces

n0(1)nψ3(2n+2)=k1(1)k1116[ψ3(k)+ψ3(k+12)\displaystyle \sum_{n\geq 0} (-1)^n\psi^3(2n+2)=\sum_{k \geq 1} (-1)^{k-1}\frac{1}{16}[\psi^3(k)+\psi^3(k+\frac{1}{2})

=3256[ζ(4,3/4)ζ(4,5/4)256+3π4]=\frac{3}{256}[\zeta(4,3/4)-\zeta(4,5/4)-256+3\pi^4]

=ψ3(1/4)512+ψ3(3/4)512+9π4256=\displaystyle \frac{-\psi^3(1/4)}{512}+\frac{\psi^3(3/4)}{512}+\frac{9\pi^4}{256}

and thus , we have π4256+ψ3(3/4)256\boxed{\frac{\pi^4}{256}+\frac{\psi^3(3/4)}{256}}

Aman Rajput - 4 years, 11 months ago

Problem 8: Show that n=01n4+4  =  18(1+πcothπ) \sum_{n=0}^\infty \frac{1}{n^4 + 4} \; = \; \tfrac18(1 + \pi \mathrm{coth}\,\pi)

This problem has been solved by Ishan Singh and Aditya Kumar.

Mark Hennings - 4 years, 11 months ago

Log in to reply

Solution To Problem 8 :

Lemma : n=01n2+1=12(1+πcoth(π))\sum_{n=0}^{\infty} \dfrac{1}{n^2+1} = \dfrac{1}{2} (1 + \pi \coth(\pi))

Proof : Let S=n=01n2+1\text{S} = \sum_{n=0}^{\infty} \dfrac{1}{n^2+1}

    S=12in=0(1ni1n+i)\implies \text{S} = \dfrac{1}{2i} \sum_{n=0}^{\infty} \left(\dfrac{1}{n-i} - \dfrac{1}{n+i}\right)

Using Digamma Regularization, we have,

S=12i[ψ(i)ψ(i)]\text{S} = \dfrac{1}{2i} [\psi(i) - \psi(-i)]

Simplifying using Digamma Reflection Formula, we have,

S=12(1+iπcot(iπ))\text{S} = \dfrac{1}{2} (1 + i \pi \cot(i\pi))

=12(1+πcoth(π)) = \dfrac{1}{2} (1 + \pi \coth(\pi)) \quad \square

Now,

J=n=01n4+4 \text{J} = \sum _{n=0}^{\infty} \dfrac{1}{n^4+4}

=n=01(n2+2n+2)(n22n+2) = \sum _{n=0}^{\infty} \dfrac{1}{(n^2+2n+2)(n^2-2n+2)}

Using Partial Fraction,

J=14n=01(n22n+2)+18n=0(n+2n2+2n+2nn22n+2) \text{J} = \dfrac{1}{4} \sum _{n=0}^{\infty} \dfrac{1}{(n^2-2n+2)} + \dfrac{1}{8} \sum _{n=0}^{\infty} \left( \dfrac{n+2}{n^2+2n +2} - \dfrac{n}{n^2-2n+2} \right)

=18+14n=11(n1)2+1+18n=0(n+2n2+2n+2nn22n+2) =\dfrac{1}{8} + \dfrac{1}{4} \sum _{n=1}^{\infty} \dfrac{1}{(n-1)^2+1} + \dfrac{1}{8} \sum _{n=0}^{\infty} \left( \dfrac{n+2}{n^2+2n +2} - \dfrac{n}{n^2-2n+2} \right)

=18+14n=01n2+1+18n=0(Tn+2Tn) =\dfrac{1}{8} + \dfrac{1}{4} \sum _{n=0}^{\infty} \dfrac{1}{n^2+1} +\dfrac{1}{8} \sum _{n=0}^{\infty} \left( T_{n+2} - T_{n} \right)

where Tn=nn22n+2T_{n} = \dfrac{n}{n^2-2n+2}

Note that the latter sum telescopes. Evaluating it, we have,

n=0(Tn+2Tn)=1\sum _{n=0}^{\infty} \left( T_{n+2} - T_{n} \right) = -1

    J=14n=01n2+1 \implies \text{J} = \dfrac{1}{4} \sum _{n=0}^{\infty} \dfrac{1}{n^2+1}

=18(1+πcoth(π)) = \dfrac{1}{8} (1 +\pi \coth(\pi)) \quad \square

Ishan Singh - 4 years, 11 months ago

Solution to Problem 8:

For any positive integer N>2N>2, let CNC_N be the positively oriented square contour with corners at ±(N+12)±i(N+12)\pm(N+\tfrac12) \pm i(N+\tfrac12). Let RR be the set {1+i,1i,1i,1+i}\{1+i,1-i,-1-i,-1+i\}. Then, since πcotπz\pi \cot\pi z is a meromorphic function, periodic of period 11, with a simple pole of residue 11 at each integer, we see that 12πiCNπcotπzz4+4dz=n=NNResz=nπcotπzz4+4+uRResz=uπcotπzz4+4=14+2n=0N1n4+4+uRResz=uπcotπzz4+4=14+2n=0N1n4+4116πuRucotπu \begin{array}{rcl} \displaystyle \frac{1}{2\pi i}\int_{C_N} \frac{\pi \cot \pi z}{z^4 + 4}\,dz & = & \displaystyle \sum_{n=-N}^N \mathrm{Res}_{z=n}\frac{\pi \cot\pi z}{z^4+4} + \sum_{u \in R}\mathrm{Res}_{z=u}\frac{\pi \cot\pi z}{z^4+4} \\ & = & \displaystyle -\tfrac14 + 2\sum_{n=0}^N \frac{1}{n^4 + 4} + \sum_{u \in R} \mathrm{Res}_{z=u} \frac{\pi \cot\pi z}{z^4 + 4} \\ & = & \displaystyle -\tfrac14 + 2\sum_{n=0}^N \frac{1}{n^4 + 4} - \tfrac{1}{16}\pi \sum_{u \in R} u \cot \pi u \end{array} Since cotπ(ϵ+ηi)  =  icothπη \cot \pi(\epsilon + \eta i) \; = \; - i \mathrm{coth}\,\pi\eta for ϵ,η{1,1}\epsilon,\eta \in \{1,-1\}, it follows that 12πiCNπcotπzz4+4dz  =  14+2n=0N1n4+414πcothπ \frac{1}{2\pi i}\int_{C_N} \frac{\pi \cot \pi z}{z^4 + 4}\,dz \; = \; -\tfrac14 + 2\sum_{n=0}^N \frac{1}{n^4 + 4} - \tfrac14\pi \mathrm{coth}\,\pi Since πcotπz\pi \cot \pi z is uniformly bounded on CNC_N for all integers NN, we deduce that limN12πiCNπcotπzz4+4dz  =  0 \lim_{N \to \infty} \frac{1}{2\pi i}\int_{C_N} \frac{\pi \cot \pi z}{z^4 + 4}\,dz \; = \; 0 which gives the result.

Mark Hennings - 4 years, 11 months ago

Claim: a=18b4a4+4b4=bπcoth(bπ)1 \displaystyle \sum_{a=1}^\infty \frac {8b^4}{a^4+4b^4} = b \pi \coth (b \pi) - 1

Proof: Read Brian Chen's comment here.

The result follows.

Pi Han Goh - 4 years, 11 months ago

S=n=01n4+4S=\sum _{ n=0 }^{ \infty }{ \frac { 1 }{ { n }^{ 4 }+4 } }

On splitting it using partial fractions, we get:

S=14i{n=01n22in=01n2+2i}S=\frac { 1 }{ 4i } \left\{ \sum _{ n=0 }^{ \infty }{ \frac { 1 }{ { n }^{ 2 }-2i } } -\sum _{ n=0 }^{ \infty }{ \frac { 1 }{ { n }^{ 2 }+2i } } \right\}

We use the identity: n=01a2+n2=1+aπcothaπ2a2\displaystyle\sum_{n=0}^\infty\frac{1}{a^2+n^2}=\frac{1+a\pi\coth a\pi}{2a^2}. See the proof here.

On simplifying (exercise to readers), we get: n=01n4+4  =  18(1+πcothπ)\boxed{\displaystyle \sum_{n=0}^\infty \frac{1}{n^4 + 4} \; = \; \tfrac18(1 + \pi\mathrm{coth}\,\pi)}

Aditya Kumar - 4 years, 11 months ago

Problem 4:

Evaluate \substack0rnr1(mod3)(nr)\large \sum_{\substack {0 \leq r \leq n \\ r \equiv 1 \pmod 3 }} {n \choose r}

This problem has been solved by Ishan Singh.

A Former Brilliant Member - 4 years, 11 months ago

Log in to reply

Solution Of Problem 4 :

Let S=\substack0rnr1(mod3)(nr)\text{S} = \sum_{\substack {0 \leq r \leq n \\ r \equiv 1 \pmod 3 }} {\dbinom{n}{r}}

Since,

r=1n(nr)xr=(1+x)n\sum_{r=1}^{n} \dbinom{n}{r} x^r = (1+x)^n

    r=1n(nr)ωr=(1+ω)n(1) \implies \sum_{r=1}^{n} \dbinom{n}{r} {\omega}^r = (1+\omega)^n \tag{1}

r=1n(nr)ω2r=(1+ω2)n(2)\sum_{r=1}^{n} \dbinom{n}{r} {\omega}^{2r} = (1 + {\omega}^2)^n \tag{2}

r=1n(nr)=2n(3) \sum_{r=1}^{n} \dbinom{n}{r} = 2^n \tag{3}

where ω=eiπ3\omega = e^{i \frac{\pi}{3}}.

Operating ω2(1)+ω(2)+(3) {\omega}^2 \cdot (1) + \omega \cdot (2) + (3) , we have,

S=13[ω2(1+ω)n+ω(1+ω2)n+2n]\text{S} = \dfrac{1}{3} \left[ {\omega}^2(1+\omega)^n + \omega (1 + {\omega}^2)^n + 2^n \right]


Similarly, we get,

Generalization : \substack0rnrk(modm)(nr)xr=r0(nrm+k)xrm+k=1mr=0m1ωrk(1+xωr)n \sum_{\substack {0 \leq r \leq n \\ r \equiv k \pmod m }} {\dbinom{n}{r}} x^r = \sum_{r \geq 0} \dbinom{n}{rm + k} x^{rm+k} = \dfrac{1}{m} \sum_{r=0}^{m-1} {\omega}^{-rk} (1 + x\omega^r)^n

where ω\omega represents one of the non real mthm^{th} roots of unity.

Ishan Singh - 4 years, 11 months ago

Problem 5:

Prove That r=2(1)rFrFr1=ϕ1 \displaystyle \sum_{r=2}^{\infty} \dfrac{(-1)^r}{F_{r} F_{r-1}} = \phi - 1

Notation : FrF_{r} denotes Fibonacci Number.

This problem has been solved my Mark Hennings.

Ishan Singh - 4 years, 11 months ago

Log in to reply

Solution to Problem 5:

Since Fn  =  15[ϕn(ϕ1)n] F_n \; =\; \tfrac{1}{\sqrt{5}}\big[\phi^n - (-\phi^{-1})^n\big] we have 1FnFn1=5(ϕn(ϕ1)n)(ϕn1(ϕ1)n1)  =  5ϕ2n1(ϕ2n(1)n)(ϕ2n2(1)n1)=5ϕ+ϕ1[1ϕ2n(1)n+1ϕ2n2(1)n1] \begin{array}{rcl} \displaystyle \frac{1}{F_n F_{n-1}} & = & \displaystyle \frac{5}{\big(\phi^n - (-\phi^{-1})^n\big)\big(\phi^{n-1} - (-\phi^{-1})^{n-1}\big)} \; = \; \frac{5\phi^{2n-1}}{\big(\phi^{2n} - (-1)^n\big)\big(\phi^{2n-2} - (-1)^{n-1}\big)} \\ & = & \displaystyle \frac{5}{\phi + \phi^{-1}}\left[ \frac{1}{\phi^{2n} - (-1)^n} + \frac{1}{\phi^{2n-2} - (-1)^{n-1}}\right] \end{array} and hence (the series telescopes): r=2(1)nFnFn1=5ϕ+ϕ1n=2[(1)nϕ2n(1)n(1)n1ϕ2n2(1)n1]=5ϕ+ϕ1×1ϕ2+1  =  5ϕ(ϕ2+1)2  =  5ϕ(ϕ+2)2=ϕϕ+1  =  ϕ1 \begin{array}{rcl} \displaystyle \sum_{r=2}^\infty \frac{(-1)^n}{F_nF_{n-1}} & = & \displaystyle \frac{5}{\phi+\phi^{-1}}\sum_{n=2}^\infty \left[ \frac{(-1)^n}{\phi^{2n} - (-1)^n} - \frac{(-1)^{n-1}}{\phi^{2n-2} - (-1)^{n-1}}\right] \\ & = & \displaystyle \frac{5}{\phi + \phi^{-1}} \times \frac{1}{\phi^2 + 1} \; =\; \frac{5\phi}{(\phi^2 + 1)^2} \; = \; \frac{5\phi}{(\phi+2)^2} \\ & = & \displaystyle \frac{\phi}{\phi+1} \; = \; \phi - 1 \end{array} as required. I am happy to let Ishan set another problem; I am very busy at present.

Mark Hennings - 4 years, 11 months ago

Problem 7:

Prove that:

n=1Hn(r)zn=Lir(z)1z\sum _{ n=1 }^{ \infty }{ { H }_{ n }^{ \left( r \right) }{ z }^{ n } } =\frac { { \text{Li} }_{ r }\left( z \right) }{ 1-z }

Here Hn(r){ H }_{ n }^{ \left( r \right) } is generalized harmonic number and Lir(z){ Li }_{ r }\left( z \right) is polylogarithm.

This problem has been solved by Mark Hennings and Deeparaj Bhat.

Aditya Kumar - 4 years, 11 months ago

Log in to reply

Solution to Problem 7:

(1z)n=1Hn(r)zn=n=1Hn(r)znn=1Hn(r)zn+1=z+n=2[Hn(r)Hn1(r)]zn  =  z+n=2znnr=Lir(z) \begin{array}{rcl} \displaystyle (1-z)\sum_{n=1}^\infty H_n^{(r)}z^n & = & \displaystyle \sum_{n=1}^\infty H^{(r)}_n z^n - \sum_{n=1}^\infty H^{(r)}_n z^{n+1} \\ & = & \displaystyle z + \sum_{n=2}^\infty \big[H^{(r)}_n - H^{(r)}_{n-1}\big]z^n \; = \; z + \sum_{n=2}^\infty \frac{z^n}{n^r} \\ & = & \displaystyle \mathrm{Li}_r(z) \end{array} for z<1|z| < 1.

Mark Hennings - 4 years, 11 months ago

Consider the following summation: (1z)n=1Hn(r)zn=n=1(Hn(r)Hn1(r))zn(letH0(r)=0)=Lir(z)Q.E.D. \begin{aligned} & (1-z) \sum_{n=1}^{\infty} H_n^{(r)} z^n \\ &= \sum_{n=1}^{\infty} (H_n^{(r)} - H_{n-1}^{(r)})z^n \quad (let \, H_0^{(r)}=0 )\\&= \text{Li}_r(z) \\ \large Q. E. D. \end{aligned}

A Former Brilliant Member - 4 years, 11 months ago

Problem 10:

Evaluate: k=1ζ(2k+1)12k+3\displaystyle \sum_{k=1}^\infty\frac{\zeta(2k+1)-1}{2k+3}

This problem has been solved by Mark Hennings.

Aditya Kumar - 4 years, 11 months ago

Log in to reply

Solution to Problem 10:

The sum is S  =  k1ζ(2k+1)12k+3  =  k1n21(2k+3)n2k+1  =  n2F(1n) S \; =\; \sum_{k \ge 1}\frac{\zeta(2k+1)-1}{2k+3} \; =\; \sum_{k \ge 1} \sum_{n \ge 2} \frac{1}{(2k+3)n^{2k+1}} \; = \; \sum_{n \ge 2} F\big(\tfrac{1}{n}\big) where F(x)  =  k1x2k+12k+3  =  12x2ln(1+x1x)13xx1x<1 F(x) \; = \; \sum_{k \ge 1} \frac{x^{2k+1}}{2k+3} \; = \; \frac{1}{2x^2}\ln\left(\frac{1+x}{1-x}\right) - \tfrac13x - x^{-1} \qquad |x| < 1 Thus S  =  n2(12n2ln(n+1n1)13nn) S \; = \; \sum_{n \ge 2} \left(\tfrac12n^2\ln\left(\frac{n+1}{n-1}\right) - \frac{1}{3n} - n\right) After a shed-load of simplification, the partial sum SN=n=2N+1(12n2ln(n+1n1)13nn)=12(N+1)2ln(N+2)+12N2ln(N+1)+2ln(G(N+1)Γ(N+1)N)12ln2+4313HN+112(N+1)(N+2) \begin{array}{rcl} S_N & = & \displaystyle \sum_{n = 2}^{N+1} \left(\tfrac12n^2\ln\big(\frac{n+1}{n-1}\big) - \frac{1}{3n} - n\right) \\ & = & \displaystyle \tfrac12(N+1)^2\ln(N+2) + \frac12N^2\ln(N+1) + 2\ln\left(\frac{G(N+1)}{\Gamma(N+1)^N}\right) \\ & & {} \displaystyle- \tfrac12\ln2 + \tfrac43 - \tfrac13H_{N+1} - \tfrac12(N+1)(N+2) \end{array} where GG is the Barnes G-function. With the known asymptotics lnG(N+1)112lnA+12Nln(2π)+12(N216)lnN34N2lnΓ(N+1)(N+1)ln(N+1)(N+1)12ln(N+1)+12ln(2π)+112(N+1) \begin{array}{rcl} \displaystyle \ln G(N+1) & \sim & \displaystyle \tfrac{1}{12} - \ln A + \tfrac12N\ln(2\pi) + \tfrac12(N^2 - \tfrac16)\ln N - \tfrac34N^2 \\ \displaystyle \ln \Gamma(N+1) & \sim & (N+1)\ln(N+1) - (N+1) - \tfrac12\ln(N+1) + \tfrac12\ln(2\pi) + \frac{1}{12(N+1)} \end{array} as NN \to \infty, where AA is the Glaisher constant, we deduce that ln(G(N+1)Γ(N+1)N)    lnA+14N(N+4)+12(N216)lnN12N(2N+1)ln(N+1) \ln\left(\frac{G(N+1)}{\Gamma(N+1)^N}\right) \; \sim \; -\ln A + \tfrac14N(N+4) + \tfrac12(N^2 - \tfrac16)\ln N - \tfrac12N(2N+1)\ln(N+1) as NN \to \infty and hence we can deduce (after even more simplification) that S  =  limNSN  =  13122lnA12ln213γ S \; = \; \lim_{N \to \infty}S_N \; =\; \tfrac{13}{12} - 2\ln A - \tfrac12\ln2 - \tfrac13\gamma

Mark Hennings - 4 years, 11 months ago

Log in to reply

Awesome solution!

Aditya Kumar - 4 years, 11 months ago

Solution To Problem 10 :

From the generating function of Riemann Zeta, we have,

n=1ζ(n+1) xn=γψ(1x) \sum_{n=1}^{\infty} \zeta(n+1) \ x^{n} = - \gamma - \psi(1-x)

    n=1(1+(1)n)ζ(n+1) xn=2γψ(1x)ψ(1+x) \implies \sum_{n=1}^{\infty} (1 + (-1)^{n}) \zeta(n+1) \ x^{n} = - 2 \gamma - \psi(1-x) - \psi(1+x)

    n=1ζ(2n+1) x2n+1=γxxψ(x)πx2cot(πx)12() \implies \sum_{n=1}^{\infty} \zeta(2n+1) \ x^{2n+1} = -\gamma x - x \psi(x) - \dfrac{\pi x}{2} \cot (\pi x) - \dfrac{1}{2} \quad \quad (*)

Now,

S=k=1ζ(2k+1)12k+3 \text{S} = \sum_{k=1}^\infty\frac{\zeta(2k+1)-1}{2k+3}

=01x[k=1(x2k+1ζ(2k+1)x2k+1)]dx = \int_{0}^{1} x \left[ \sum_{k=1}^{\infty} ( x^{2k+1} \zeta(2k+1) - x^{2k+1} ) \right] \mathrm{d}x

Using ()(*), we have,

S=01(γx2+x2ψ(x)+x2+πx22cot(πx)+x41x2)dx \text{S} = - \int_{0}^{1} \left( \gamma x^2 + x^2 \psi (x) + \dfrac{x}{2} + \dfrac{\pi x^2}{2} \cot(\pi x) + \dfrac{x^4}{1-x^2} \right) \mathrm{d}x

=13122logA12log2γ3 = \dfrac{13}{12} -2\log A - \dfrac{1}{2} \log 2 - \dfrac{\gamma}{3} \quad \square

Ishan Singh - 4 years, 11 months ago

Problem 14:

Prove that:

n=1ψ1(n)2nn2=19π41440π224log2(2)log4(2)24ζ(3)log(2)4\sum_{n=1}^\infty\frac{\psi_1(n)}{2^n n^2}=\frac{19\pi^4}{1440}-\frac{\pi^2}{24}\log^2(2)-\frac{\log^4(2)}{24}-\frac{\zeta(3) \log(2)}{4}

This problem has been solved by Mark Hennings.

Aditya Kumar - 4 years, 11 months ago

Log in to reply

Solution to Problem 14:

We note that n=1ψ(1)(n)2nn2=12ζ(2)n=112n+1(n+1)2[Hn(2)ζ(2)]=ζ(2)n=112nn2n=1Hn(2)2n+1(n+1)2=ζ(2)[112π212ln22]n=1Hn(2)(n+1)22n+1 \begin{array}{rcl} \displaystyle \sum_{n=1}^\infty \frac{\psi^{(1)}(n)}{2^n n^2} & = & \displaystyle \tfrac12\zeta(2) - \sum_{n=1}^\infty \frac{1}{2^{n+1}(n+1)^2}\big[H^{(2)}_n - \zeta(2)\big] \\ & = & \displaystyle\zeta(2)\sum_{n=1}^\infty \frac{1}{2^n n^2} - \sum_{n=1}^\infty \frac{H_n^{(2)}}{2^{n+1}(n+1)^2} \\ & = & \displaystyle\zeta(2)\Big[ \tfrac{1}{12}\pi^2 - \tfrac12\ln^22\Big] - \sum_{n=1}^\infty \frac{H_n^{(2)}}{(n+1)^2 2^{n+1}} \end{array} Going back to the previous problem, we have A(x)=n=1(Hn(2))2xn  =  n=1(Hn1(2)+1n2)2xn=n=1(Hn(2))2xn+1+2n=1Hn(2)(n+1)2xn+1+Li4(x)=xA(x)+2n=1Hn(2)(n+1)2xn+1+Li4(x)(1x)A(x)=2n=1Hn(2)(n+1)2xn+1+Li4(x) \begin{array}{rcl} \displaystyle A(x) & = & \displaystyle \sum_{n=1}^\infty \big(H_n^{(2)}\big)^2x^n \; =\; \sum_{n=1}^\infty \left(H_{n-1}^{(2)} + \tfrac{1}{n^2}\right)^2 x^n \\ & = & \displaystyle \sum_{n=1}^\infty \big(H_n^{(2)}\big)^2 x^{n+1} + 2\sum_{n=1}^\infty \frac{H_n^{(2)}}{(n+1)^2} x^{n+1} + \mathrm{Li}_4(x) \\ & =& \displaystyle xA(x) + 2\sum_{n=1}^\infty \frac{H_n^{(2)}}{(n+1)^2} x^{n+1} + \mathrm{Li}_4(x) \\ (1 - x)A(x) & = & \displaystyle 2\sum_{n=1}^\infty \frac{H_n^{(2)}}{(n+1)^2} x^{n+1} + \mathrm{Li}_4(x) \end{array} and hence, putting x=12x=\tfrac12, 12n=1(Hn(2))22n  =  2n=1Hn(2)(n+1)22n+1+Li4(12) \tfrac12\sum_{n=1}^\infty \frac{\big(H_n^{(2)}\big)^2}{2^n} \; = \; 2\sum_{n=1}^\infty \frac{H_n^{(2)}}{(n+1)^2 2^{n+1}} + \mathrm{Li}_4(\tfrac12) and so n=1Hn(2)(n+1)22n+1  =  11440π4124π2ln22+124ln42+14ζ(3)ln2 \sum_{n=1}^\infty \frac{H_n^{(2)}}{(n+1)^2 2^{n+1}} \; = \; \tfrac{1}{1440}\pi^4 -\tfrac{1}{24}\pi^2\ln^22 + \tfrac{1}{24}\ln^42 + \tfrac14\zeta(3)\ln2 so that n=1ψ(1)(n)2nn2  =  191440π4124π2ln22124ln4214ζ(3)ln2 \sum_{n=1}^\infty \frac{\psi^{(1)}(n)}{2^n n^2} \; = \; \tfrac{19}{1440}\pi^4 - \tfrac{1}{24}\pi^2\ln^22 - \tfrac{1}{24}\ln^42 - \tfrac14\zeta(3)\ln2 Someone else can post the next question.

Mark Hennings - 4 years, 11 months ago

Log in to reply

This question was essentially the same as the previous question. @Aditya Kumar If you set the next question, please post a new one.

Ishan Singh - 4 years, 11 months ago

Log in to reply

@Ishan Singh Can you post the problem? Post it in this note as it will be the 15th problem (don't count spl prob).

Aditya Kumar - 4 years, 11 months ago

Problem 15:

Prove that:

n=11(n2+1)2=π4(coth(π)+πcsch2(π)2π)\displaystyle \sum_{n=1}^{\infty}\frac{1}{(n^{2}+1)^{2}}=\frac{\pi}{4}\left(\coth(\pi)+\pi \text{csch}^{2}(\pi)-\frac{2}{\pi}\right)

Aditya Kumar - 4 years, 11 months ago

Log in to reply

Solution to Problem 15:

Refer to my solution to Problem 8 for some of the technical details required about CNC_N and πcotπz\pi \cot \pi z.

Integrating πcotπz(z2+1)2 \frac{\pi \cot \pi z}{(z^2+1)^2} around the positively-oriented square contour CNC_N with corners at ±(N+12)±i(N+12)\pm(N+\tfrac12) \pm i(N+\tfrac12), we deduce that 12πiCNπcotπz(z2+1)2dz=(n=NNResz=n+Resz=i+Resz=i)πcotπz(z2+1)2dz=1+2n=1N1(n2+1)2+(Resz=i+Resz=i)πcotπz(z2+1)2\begin{array}{rcl} \displaystyle \frac{1}{2\pi i}\int_{C_N}\frac{\pi \cot \pi z}{(z^2+1)^2}\,dz & = & \displaystyle \left(\sum_{n =-N}^N\mathrm{Res}_{z=n} + \mathrm{Res}_{z=i} + \mathrm{Res}_{z=-i}\right) \frac{\pi \cot\pi z}{(z^2 + 1)^2}\,dz \\ & = & \displaystyle 1 + 2\sum_{n=1}^N \frac{1}{(n^2+1)^2} + \left(\mathrm{Res}_{z=i} + \mathrm{Res}_{z=-i}\right)\frac{\pi \cot\pi z}{(z^2 + 1)^2} \end{array} and Resz=±iπcotπz(z2+1)2  =  ddzπcotπz(z±i)2z=±i  =  14π2cosech2π14πcothπ \mathrm{Res}_{z=\pm i}\frac{\pi \cot \pi z}{(z^2+1)^2} \; = \; \frac{d}{dz} \frac{\pi \cot \pi z}{(z \pm i)^2} \Big|_{z=\pm i} \; = \; -\tfrac14\pi^2 \mathrm{cosech}^2\pi - \tfrac14\pi\mathrm{coth}\pi so that 12πiCNπcotπz(z2+1)2dz  =  1+2n=1N1(n2+1)212π2cosech2π12πcothπ \frac{1}{2\pi i}\int_{C_N} \frac{\pi \cot \pi z}{(z^2+1)^2}\,dz \; = \; 1 + 2\sum_{n=1}^N \frac{1}{(n^2+1)^2} - \tfrac12\pi^2 \mathrm{cosech}^2\pi - \tfrac12\pi\mathrm{coth}\pi Since the integral around CNC_N tends to 00 as NN \to \infty, the result follows.

As before, I pass on setting the next problem.

Mark Hennings - 4 years, 11 months ago

Log in to reply

Alternate Method:

n=11(n2+1)2=14n=1{1(1+in)211+in+1(1+in)2+11+in}\displaystyle \sum_{n=1}^\infty \frac{1}{(n^2+1)^2}=\frac{1}{4}\sum_{n=1}^\infty \left\{ \frac{1}{(-1+in)^2}-\frac{1}{-1+in}+\frac{1}{(1+in)^2}+\frac{1}{1+in}\right\}

Here, we can use polygamma function and use their reflection formulas to get to the final result.

Aditya Kumar - 4 years, 11 months ago

Sir I can't think of a good problem and Ishan is down with fever. Can you post the next problem here?

Aditya Kumar - 4 years, 11 months ago

The next problem would be posted here.

Aditya Kumar - 4 years, 11 months ago

PROBLEM 16: If 0<θ<2π0 < \theta < 2\pi, evaluate n=1cosnθn \sum_{n=1}^\infty \frac{\cos n\theta}{n} Don't forget to justify the convergence!

Mark Hennings - 4 years, 11 months ago

Log in to reply

Solution To Problem 16 :

Let an=1na_{n} = \dfrac{1}{n} and bn=cosnθb_{n} = \cos n \theta. Note that both sequences satisfy the criterion of Dirchlet's Test. Therefore, the sum S=n=1cosnθn \displaystyle \text{S} = \sum_{n=1}^{\infty} \dfrac{\cos n \theta}{n} converges.

Now,

S=n=1cosnθn \text{S} = \sum_{n=1}^{\infty} \dfrac{\cos n \theta}{n}

=(n=1einθn) = \Re \left( \sum_{n=1}^{\infty} \dfrac{e^{i n \theta}}{n} \right)

=(ln(1eiθ)) = -\Re \left(\ln (1-e^{i \theta}) \right)

=ln2sin(θ2) = - \ln\left| 2\sin \left( \dfrac{\theta}{2} \right) \right| \quad \square

Ishan Singh - 4 years, 11 months ago

The contest has shifted to this note.

Aditya Kumar - 4 years, 11 months ago

Problem 9 :

Evaluate

n=1(Hn)22n\sum_{n=1}^{\infty} \dfrac{(H_{n})^2}{2^n}

Notation : HnH_{n} denotes the Harmonic Number.

This problem has been solved by Aditya Kumar.

Ishan Singh - 4 years, 11 months ago

Log in to reply

Solution to Problem 9:

I'll use the generating function: log2(1t)+Li2(t)(1t)=n=1Hn2tn\frac{\log^{2}(1-t)+Li_{2}(t)}{(1-t)}=\sum_{n=1}^{\infty}H_{n}^{2}t^{n}

Substitute: t=12t=\frac{1}{2}.

Hence the final answer is: (ln2)2+π26\boxed{{ \left( \ln { 2 } \right) }^{ 2 }+\frac { { \pi }^{ 2 } }{ 6 } }

Aditya Kumar - 4 years, 11 months ago

Log in to reply

Proof For Generating Function :

f(x)=n=1Hn2xn f(x) = \sum_{n=1}^{\infty} H^2 _{n} x^n

Since Hn=Hn1+1n    Hn2=Hn12+1n2+2Hn1nH_n = H_{n-1} +\dfrac{1}{n} \implies H^2 _{n} = H^2 _{n-1} +\dfrac{1}{n^2} + 2\dfrac{H_{n-1}}{n} and we have,

f(x)=n=1Hn2xn=n=1(Hn12xn+xnn2+2Hn1nxn)f(x) = \sum_{n=1}^{\infty} H^2 _{n} x^n = \sum_{n=1}^{\infty} \left(H^2 _{n-1} x^{n} +\dfrac{x^n}{n^2} + 2\dfrac{H_{n-1}}{n} x^n\right)

Changing summation index n1n n-1 \mapsto n in the first and third sum, we have,

f(x)=xn=1Hnxn+Li2(x)+2n=1Hnn+1xn+1 f(x) = x\sum_{n=1}^{\infty} H_{n} x^n + \operatorname{Li}_{2}(x) + 2\sum_{n=1}^{\infty} \dfrac{H_{n}}{n+1} x^{n+1}

    f(x)=xf(x)+Li2(x)+2n=1Hnn+1xn+1\implies f(x) = xf(x) + \operatorname{Li}_{2}(x) + 2\sum_{n=1}^{\infty} \dfrac{H_{n}}{n+1} x^{n+1}

Also,

n=1Hnn+1xn+1=0xn=1Hntndt \sum_{n=1}^{\infty} \dfrac{H_{n}}{n+1} x^{n+1} = \int_{0}^{x} \sum_{n=1}^{\infty} H_{n} t^{n} \mathrm{d}t

=0xln(1t)1tdt = -\int_{0}^{x}\dfrac{\ln(1-t)}{1-t} \mathrm{d}t

=ln2(1x)2 = \dfrac{\ln^2(1-x)}{2}

    f(x)=xf(x)+Li2(x)+ln2(1x)\implies f(x) = xf(x) + \operatorname{Li}_{2}(x) + \ln^2(1-x)

    f(x)=ln2(1x)+Li2(x)1x \implies f(x) = \dfrac{\ln^2(1-x) + \operatorname{Li}_{2}(x)}{1-x}

Ishan Singh - 4 years, 11 months ago
×

Problem Loading...

Note Loading...

Set Loading...